A recent survey found that more computers than copies of com

This topic has expert replies
Master | Next Rank: 500 Posts
Posts: 160
Joined: Tue Jul 07, 2009 1:09 pm
Thanked: 1 times
Followed by:1 members

by Sharma_Gaurav » Sun Feb 12, 2012 9:52 am
OA is C

option A and B has this word " few " which confused me initially to assume that means most computer users illeglaly dump computer programes. Hence i made mistake.

I agree with original answer now. Thanks to TestLuv for making it clear.

User avatar
Legendary Member
Posts: 698
Joined: Tue Jul 21, 2015 12:12 am
Location: Noida, India
Thanked: 32 times
Followed by:26 members
GMAT Score:740

by richachampion » Mon Aug 03, 2015 12:09 am
Testluv wrote:--editing my post to clarify in response to recent pm--

The correct answer is definitely choice C. Work with the question stem here. Because this is an EXCEPT question, the four wrong answers counter objections against the argument. The right answer will be something that will not counter an objection.

Fact (phenomenon): Germans buy more computers than they do computer programs.

Author's explanation: there's a lot of illegal copying of computer programs going on.

So, the author thinks that because they buy more computers than computer programs, the Germans are illegally copying computer programs. Objections would point to alternative explanations. For examples:

--Germans write their own programs
--computer programs are available for free
--Germans don't want/need computer programs
--Germans legally (rather than illegally) copy the computer programs

The wrong answers will counter objections such as those listed above while the right answer will either fail to counter objections such as these or will itself raise an objection. Choice A counters the first objection listed above, choice B the second, choice D the third, and choice E the last.

Choice C is tricky because it seems to counter the objection that Germans were purchasing computer programs outside of Germany. However, where they purchase the computer programs is outside the scope of the passage. (When I got to this choice, I had to recheck the passage before I selected this choice).

Maihuna, what's the source of this one? I'm guessing LSAT. Such tricky question stems are pretty uncommon (though still possible) on the GMAT.
This is from gmat paper based test.

Newbie | Next Rank: 10 Posts
Posts: 4
Joined: Wed Aug 12, 2020 11:37 am
Eliminate four out of five

Option (C) : Falls outside the jurisdiction - Correct Answer

Other Options : Weaken the Argument